LSAT and Law School Admissions Forum

Get expert LSAT preparation and law school admissions advice from PowerScore Test Preparation.

 Administrator
PowerScore Staff
  • PowerScore Staff
  • Posts: 8916
  • Joined: Feb 02, 2011
|
#23962
Complete Question Explanation

Strengthen-CE. The correct answer choice is (D)

This is another Resolve the Paradox question. We already know from the stimulus that if we compare consumption of milk with that of an equivalent amount of oil-based coffee lighteners, a tablespoon of non-dairy creamer increases one’s cholesterol level more than a teaspoon of whole milk. Now the manufacturers of these non-diary lighteners make the claim that the cholesterol level of the typical consumer is increased more by milk than by non-dairy lighteners.
  • Premise (from the stimulus): ..... One tablespoon of a coconut oil-based coffee lightener raises one’s blood cholesterol level to a higher level than does one tablespoon (an “identical amount”) of whole milk.

    Premise (from the text of question 14): ..... Manufacturers claim that oil based lighteners increase the typical consumer’s cholesterol level less than whole milk.
How can we resolve these two premises, which seem to contradict one another? We need to find another premise which reconciles the two (notice the subtle differences between the information from the stimulus and the claims of the manufacturers: the stimulus discusses equal amounts of whole milk and oil-based creamers, and the manufacturers’ claims are based instead on “typical” consumption).

Answer choice (A): This answer is incorrect. The stimulus discusses the specific and immediate effects of consumption of these lighteners on one’s blood cholesterol level. This answer choice discusses average cholesterol levels of a very specific group: those who consume nondairy lighteners, avoid other high-cholesterol foods, and exercise. This does not bring us any closer to a resolution of this question’s paradox.

Answer choice (B): This answer is also incorrect. The manufacturers compare typical consumption of milk with that of nondairy lighteners, and each product’s relative effects on cholesterol levels. Since both of these products are used in coffee, the information in this answer choice does nothing to distinguish one type of lightener from the other.

Answer choice (C): This answer is incorrect. Since the relevant comparison in this case is between whole milk and coconut oil based lighteners, information about a third product is irrelevant to the inquiry, and certainly doesn’t help us to resolve the apparent paradox.

Answer choice (D): This is the correct answer choice. If the typical consumer uses smaller quantities of coconut oil based lighteners than of whole milk, this reconciles the two premises listed above: even though a tablespoon of coconut oil based creamer might have more profound effects on one’s cholesterol level than the same amount of whole milk, consumers typically use substantially less of the product.

Answer choice (E): This answer is incorrect. We already know that both milk and oil-based nondairy creamers affect consumers’ blood cholesterol levels, so consumer perceptions don’t seem relevant to this discussion.
 Arindom
  • Posts: 76
  • Joined: Apr 11, 2016
|
#23190
Hi,

So, basically , the manufacturer's of coconut oil based coffee lighteners say that these lighteners do not raise the blood cholesterol level as much as a a whole milk lighteners do.

Since this is a question regarding which answer would most support the manufacturers' claim - I was between E and D but ended up picking D because E is about what consumers are convinced about and so what? the argument is about the manufacturers' claims. Upon reflection, I see why D is correct - that consumers use substantially less of these lighteners in their coffee than whole milk lighteners. So, if they are using substantially less than in any case the blood cholesterol levels will also be low. Is this analysis correct?

Thanks,

- Arindom
 Claire Horan
PowerScore Staff
  • PowerScore Staff
  • Posts: 408
  • Joined: Apr 18, 2016
|
#23226
Hi Arindom,

Yes, your analysis is correct. In terms of attacking the problem, as you read actively you will notice that there seems to be a paradox. Underline the important terms to compare them more easily. I would have underlined "identical amounts" and "usually cause the typical consumer" and tried to pre-phrase the possible differences between these terms before looking at the answer choices. After doing so, Answer D pops out as the correct answer because it resolves the paradox between these two terms.

-Claire
PowerScore LSAT and GRE Instructor
 salmach
  • Posts: 12
  • Joined: Aug 11, 2017
|
#38907
Hi powerscore,

I found this stimulus in my logical reasoning - double trouble homework. The first question was the resolve the paradox one and I had anticipated either a MBT or Resolve the Paradox question because there was no conclusion in the stimulus.

The second question's stimulus claimed that "manufacturers of coffee lighteners based on coconut oil claim that their products usually cause the typical consumer's blood cholesterol to rise to a lower level than does the use of whole milk as a lightener. Which one of the following, if true, provides the most support for the manufacturers' claim?"

I incorrectly assumed that this was a MBT question. But I realized afterwards that the answer choices must be taken as true - indicating that it was not part of the first family prove, rather that it was a strengthen question. Does this mean that the stimulus claim is somewhat of a conclusion to the first part of the question?

I hope I haven't confused you too much. Any help would be greatly appreciated.
 Eric Ockert
PowerScore Staff
  • PowerScore Staff
  • Posts: 164
  • Joined: Sep 28, 2011
|
#39416
While the lack of a conclusion in a stimulus may be an indicator that you will see either MBT, Resolve, or Cannot Be True, you don't want to assume too much. Ultimately, you have to react to the language of the question stem to truly dictate what task you are being asked to perform.

In the question you cite, the stem directs you to accept the answers as true, and identify which one supports the stimulus. That upward logical flow, where the answer supports the stimulus, is classic Strengthen language.

And while the stimulus did not present an argument, the long-winded question stem here introduced an argument ("Manufacturers....claim") based around the information in the stimulus. This is the argument you are being asked to Strengthen.

Hope that helps!

Get the most out of your LSAT Prep Plus subscription.

Analyze and track your performance with our Testing and Analytics Package.